pleaseee helpppp. is this right ???

Pleaseee Helpppp. Is This Right ???

Answers

Answer 1

Answer: Yes

Step-by-step explanation:

Answer 2
Answer: yes it is correct because they ARE interior angles on the same side of the transversal

Related Questions

Help please and thank you

Answers

Answer:

2) C

3) D

4) D(5,-5)   E(2,-7)  F(4,-1)   G(8,-8)

Step-by-step explanation:

need more points have a nice day :)

can someone please do this :// I need B done on Question 9 but I'm so mad because no one is giving me a correct fkin answer

Answers

Should be 4 for B because 8-4 would equal 4

Answer:

9.

4= length A

3= width B

B. The volume is 72

Step-by-step explanation:

I got 72 by making the whole shape into 2 different shapes and finding the volume of those first. I am not sure if this is correct but I hope it is and I hope this helps! :)

For a science experiment, Annie removes a cold liquid from a refrigerator and measure it's temperature every 1/2 minute. Annie finds that the temperature increase by
1 and 3/4 degrees Fahrenheit. What is the rate per minute of the temperature increase?

Answers

Answer:

2 1/2

Step-by-step explanation:

1/2 minute = 1 3/4 degrees fahrenheit.

1 minute = 2 1/2 degrees Fahrenheit.

you can take 1 3/4 + 1 3/4 and get the Answer.

Plz answer this question

Answers

48 because when you add it to 4 and another 5 they are talking about times so 4x5=20 then you add 20 again to the answer then add 8
48 is your answer to the problem

At a party, everyone shook hands with everybody else. There were 66 handshakes. How many people were at the party?

Answers

There would be 12 people at the party!

In general, with n+1 people, the number of handshakes is the sum of the first n consecutive numbers: 1+2+3+ ... + n.

Since this sum is n(n+1)/2, we need to solve the equation n(n+1)/2 = 66.

This is the quadratic equation n2+ n -132 = 0. Solving for n, we obtain 11 as the answer and deduce that there were 12 people at the party.

Since 66 is a relatively small number, you can also solve this problem with a hand calculator. Add 1 + 2 = + 3 = +... etc. until the total is 66. The last number that you entered (11) is n.

Hope this helps!

Answer:

Uhm. You didnt get this from me, But If there was 66 handshakes, and each person shakes another persons hand, all you gotta do is divide it by 2 and you get 66.

Step-by-step explanation:

Im right ik ty ;)

This is the angle similarity of triangles btw. I will give brainliest to whoever finds the missing angle measures.

Answers

1: 64
2: 59
3: 49
A triangle should always add up to 180 degrees. So just add up the angles you have and subtract them from 180
ABC is 64
DEF is 59
GHI is 49

Express 18√8 in the form a√b where a and b are integers and b is as small as possible.


Hint(s): Factor sqrt(8) as the product of two square roots, one of which is the square root of a perfect square.


PLEASE HELP!!!

Answers

36 square root of 2 I believe.
j. Juju told investigators that he would have a little girl in his life he

2 numbers that when added equals 36 but when multiplied equals -12

Answers

Answer:

-6 and -6

Step-by-step explanation:

-6+(-6)=-12

-6x-6=36

Answer:-6 and -6

Step-by-step explanation:

Only number 4 ‍ ‍ ‍ ‍ ‍ ‍ ‍ ‍ ‍ ‍ ‍ ‍

Answers

What u did it switch them then just times it

Sensational Cell Phones offers a plan of $30 for the first 300 minutes, and an additional $0.30 for every minute over 300. Let t represent the total talk time in minutes.
Write a piecewise-defined function that represents the cost of S(t) in terms of the number of minutes t that is used.

Answers

Answer: Sensational Cell Phones offers a plan of $30 for the first 300 minutes and an additional $0.30 for every minute over 300.

Step-by-step explanation:

Fill in the blank with the correct number.

A number is divisible by _________ if the sum of its digits is divisibly by three.

A.) 2
B.) 3
C.) 9
D.) Both 3 and 9

Answers

Answer:D

Step-by-step explanation:

The correct Answer is D

Each blank can be represented by a blank on the coordinate plane

Answers

is this the question

Step-by-step explanation:

Answer:

wat dioes this mean

Step-by-step explanation:

HELP ME IF YOU DO WILL GIVE YOU 5 STARS< A THANKS< BRAINLIEST :)

Answers

Answer:

look below

Step-by-step explanation:

Monday: answers are C and D  

Tuesday: answers are B, D, and E

Wednesday: answer is -150 times 3 = - 450

Thursday: I cant rlly show my work on the pc but the answer is 0.8125 and no it is not a terminating decimal.

Friday: answers are D and F

hope this helps

What is 46 + 47 + 48 + 49 + 50 + 51 + 52 + 53 + 54 = ?

Answers

Answer:  

450  

Step-by-step explanation:  

46 + 54 = 100  

47 + 53 = 100  

48 + 52 = 100  

49 + 51 = 100  

100 + 100 + 100 + 100 + 50 = 450

When you view the pendulum’s swing, it shows that at the very top of the swing KE = 0. What does that tell you about the pendulum’s motion at that point?
Answer: Sample Response: If KE = 0, the pendulum is not moving. At the top of the swing, the pendulum must stop for a brief moment.
What you need to include:
Zero kinetic energy means the pendulum is not moving.
The pendulum stops at the top of the swing.

Answers

Answer:

The kinetic energy of an object is given by: Kinetic energy = 1/2 * mass * velocity If the kinetic energy of the pendulum at the top of its swing is 0, then this tells us that the velocity of the pendulum at that point is 0

Answer:

If KE = 0, the pendulum is not moving. At the top of the swing, the pendulum must stop for a brief moment.

Step-by-step explanation:

PLEASE HELP WITH MATH ASAP!!!!! ITS DUE IN 10 MINS!!! ILL GIVE BRAINLIEST

Answers

Answer:

1. 37

2. Supplementary

3. Complementary

4. 17

5. 22

Step-by-step explanation:

Mrs. Raymond purchased a rectangular toy chest. Its dimensions are 4 1/2 feet by 2 1/2 feet by 3 feet. What is the total volume of the toy chest?

Answers

Answer:

33 3/4 feet cubed

Step-by-step explanation:

4 1/2 x 2 1/2 x 3

1) A train travels 25 miles in 1/5 of an hour whats the trains speed in miles per hour?
2) The height of a stack of books, h inches, is directly proportional to the number of books, n. The height of a stack of 10 books is 12 inches. A-find the constant of proportionality. B-write and equation relating to h and n. C-find the height of a stack of 24 books.
3) An 8-ounce serving of milk provides 30% of the daily value of calcium. How much milk would you have to drink to get 100% of your daily value of calcium
PLEASE HELP I NEED THESE ANSWERS FASTT

Answers

Answer:

Hope this helps! ↓

Step-by-step explanation:

1.) 90 Km/h. Example: What is the speed of a cheetah that travels 112.0 meters in 4.0 seconds? ... Speed of the cheetah in miles per hour 28m 1 mile 3600 s 63 miles

2.) y=kx is the direct variation formula. Inserting the information and variables from this scenario we get h = kn.

12 = k*(10) First solve for 'k', the constant of proportionality by dividing by 10.

1.2 = k

Next write the equation using this k and the h & n.

h = 1.2*n

Now we want to find the height if we have 24 books so plug 24 in for n.

h = 1.2 * 24

h = 28.8 inches

3.) 10/3

It’s 3 step by step because follow me to get free point

Game city charges a fee of $10 plus 3.50 per game rented. gaming central charges a fee of 22 plus 2.50 per game. For how many game rentals will the cost be the same at both stores? what is the cost?

Answers

City: 10 + 3.5g
Central: 22 + 2.5g

10 + 3.5g = 22 + 2.5g
g = 12

12 game rentals

10 + 3.5(12) = 52
22 + 2.5(12) = 52

PLEASE HELP ME WITH MY MATH ;-; if you help I will give you 5 stars, a thanks, and a brainliest

Answers

x=12
y=10
cause i solved it and found that answers

Giving Brainliest

Solve for X

Answers

Answer:

x = 10

Step-by-step explanation:

108 = (5x + 47) + (2x + 63)

5x + 47 + 2x + 63 = 180

Simplify

7x + 110 = 180

Move all terms not containing  x  to the right side of the equation.

7x = 70

Divide

x = 10

good luck, hope this helps :)

the answer to this question is 10

free brainlist. Have a great weekend

Answers

MEMEMEMEMEMEKEMEnfnff
You too ! Thank youuuu!!!!

PLEASE HELP QUICK!!!
1/7(105 + r) = 20
Which describes the solution, r, to this equation?
A. The solution is 5, which represents the number of miles she runs each day.
B, The solution is 5, which represents the number of miles she runs each week.
c. The solution is 35, which represents the number of miles she runs each day.
D. The solution is 35, which represents the number of miles she runs each week.

Answers

D. The solution is 35, which represents the number of miles she runs each week.
D. The answer is D.

HELPPPPP ASAPPPPPP PLSSSS ILL PICK BRAINLIEST FOR BEST ANSWER WITH BEST DETAILS AND STUFF

Which of the following statements shows a characteristic of a statistical question?


A:The question is focused.

B: The distribution can be broken into categories.

C:The question asks for a quantitative response.

D:The question allows for surveys to be conducted.

Answers

Answer:

A

Step-by-step explanation:

cause all auestions have to have focus on them

D the question allows for surveys

Which expression can be used to determine the total weight of b baseballs that weigh 5.25 ounces each and s softballs that weigh 6.5 ounces each?

A 5.25b +6.58
B 6.5b +5.258
C 5.25b - 6.58
D 11.75 (b + 8)

Answers

Answer:

A. 5.25b + 6.58

Step-by-step explanation:

if we had for example 5 baseballs it would be 5 times 5.25 so we have b therefore it is b times 5.25

Answer:

Its A

Step-by-step explanation:

5.25b + 6.5s

if we had for example 5 baseballs it would be 5 times 5.25 so we have b therefore it is b times 5.25

please help I'll give brainiest

Answers

What do you need help on exactly?
haha sorry if i’m wrong but i think that’s what the answer is :))

Find the value of the power.

(2/3) to the power of 3

Answers

Answer:

8/27 it's the answer

Step-by-step explanation:

(2/9)^3 = 2*2*2/3**3*3

= 8/9

Simplify the expression:
3k2+6k2–9k2+6k2

Answers

I believe that it is 6k^2!!!!
6k^2 is the simplified answer

Solve for a.

2 + 2a ≥ 6

Answers

[tex]2 + 2a \geqslant 6[/tex]

[tex]2 - 2 + 2a\geqslant 6 - 2[/tex]

[tex]2a \geqslant 4[/tex]

[tex] \frac{2a}{2} \geqslant \frac{4}{2} [/tex]

[tex]a \geqslant 2[/tex]

Answer:

Isolate the variable by dividing each side by factors that don't contain the variable

Interval Notation:

2,ထ

Inequality Form:

a≥2

Step-by-step explanation:

please help due in 5 minutes I know its a long question but I will give brainliest.

Answers

Answer:

B

Step-by-step explanation:

B is the correct answer
Other Questions
= 28.16Riya buys 9 square paving slabs, each with a side of 1m. She lays them in the form of square.(b) Shari does not like her arrangement. She gets her to lay them out like a cross. What isthe perimeter of her arrangement? What do majority rules mean Question 2 (1 point)If 5 hours is 25% of the time a trip will take, how long will it take? 2 examples of group communication? The sum of 2 numbers is 5. The larger number exceeds twice the smaller number by 14. Find thenumbers. Brandon has a board that is 10 feet long. He wants to cut the board into 4 shelves. The lengths of 3 shelves are shown in the table. How long will the fourth shelf be? Ill give u the brainlest if u are correct please hurry and show how u got ur answer 1. While doing construction on your new house, you notice that it takes 6 minutes to put up 22 shingles on the roof. Which one of the following is a correct sentence to describe the ratio of minutes to shingles?Group of answer choicesThere are 3.7 minutes for every 1 shingle put on the roof.For every 3 minutes, there are 11 shingles put up.There is 1 minute for every 0.27 of a shingle.It takes 3.7 minutes to put up 22 shingles.2..A researcher decides to study study frogs in a local forest. Funding for his project says:2.For every 8 hours in the office, you will be allowed 100 minutes in the forest.Which of the following is the correct ratio of forest to office time?Group of answer choices4.80.208312.50.083.Which of the following is equivalent to the ratio 5?You don't have to show work for this problem.Group of answer choices1:55 to 11/50.24.Consider the number: 105,279,083,000Which digit is in the ten billions place?You don't have to show work for this problem.Group of answer choices75905.Write the following number in correct scientific notation: 13.8 billionYou don't need to show work for this problem, but it's a good idea.Group of answer choices1.38 x 10^-101.38 x 10^1013.8 x 10^91.38 x 10^7 How can we avoid water pollution Find the value of x. Explain your work please.Concurrency of Angle Bisectors Please help with this question! broooooooo help me wit this plzz man Problem 44. The sum of a certain quantity "p" together with two-third of that number, and one-third of that number, becomes 234. What is the quantity "p"? (HINT: Write the equation out and solve) What is the net force in this image? a8000 N right b5000 N right c4000 N right d4000 N left e5000 N left f8000 N left Which of the following English words contains the sound closest to the Spanish E?a.endb.evilc.awesomePlease select the best answer from the choices providedABC What is the standard form of 17 and 223,000 1 It's a fact that every writer needs a good editereditor editor edited Help me! Pleaseeeeeeeeeeeeee lowest common factor of 72 and 108 What Is the author purpose in the article Miss Me 1. What state capital is at 45N and 93W?a. Austin, TXc. Sacramento, CAb. Helena, MTd. St. Paul, MN